Você está na página 1de 22

7

Chapter 4

Continuous-Time Fourier Transform


(Chapter 5)

5.1 Using the Fourier transform analysis equation, find the Fourier transform of each of the following signals:
(a) x(t) = Aδ (t − t0 ) where t0 and A are real and complex constants, respectively;
(b) x(t) = rect(t − t0 ) where t0 is a constant;
(c) x(t) = e−4t u(t − 1);
(d) x(t) = 3[u(t) − u(t − 2)]; and
(e) x(t) = e−|t| .

Solution.

(d) Let X(ω ) denote the Fourier transform of x(t). From the Fourier transform analysis equation, we can write

Z ∞
X(ω ) = 3[u(t) − u(t − 2)]e− jω t dt
−∞
Z ∞
=3 [u(t) − u(t − 2)]e− jω t dt
−∞
Z 2
=3 e− jω t dt
0
i 2
− jω t
h
1
=3 − jω e


0

= −3jω e− jω t 0
  2

= ωj3 e− j2ω − 1
 

j3 − jω − jω
= ω [e ][e − e jω ]
j3 − jω
= ωe [−2 j sin ω ]
6 − jω
= ωe sin ω
− jω
= 6e sinc ω .

Version: 2014-05-08 Copyright 2014


c Michael D. Adams
8 Continuous-Time Fourier Transform (Chapter 5)

(e) Let X(ω ) denote the Fourier transform of x(t). From the Fourier transform analysis equation, we have
Z ∞
X(ω ) = e−|t| e− jω t dt
−∞
Z 0 Z ∞
= e−|t| e− jω t dt + e−|t| e− jω t dt
−∞ 0
Z 0 Z ∞
= et e− jω t dt + e−t e− jω t dt
−∞ 0
Z 0 Z ∞
= e(1− jω )t dt + e(−1− jω )t dt
−∞ 0
i 0 i ∞
= 1−1jω e(1− jω )t − 1+1jω e(−1− jω )t
h h

−∞ 0
1 1
= 1− jω [1 − 0] − 1+ jω [0 − 1]
1 1
= 1− jω + 1+ jω
1 + jω + 1 − jω
=
(1 + jω )(1 − jω )
2
= .
1 + ω2

5.2 Use a Fourier transform table and properties of the Fourier transform to find the Fourier transform of each of
the signals below.
(a) x(t) = cos(t − 5);
(b) x(t) = e− j5t u(t + 2);
(c) x(t) = [cost]u(t);
(d) x(t) = 6[u(t) − u(t − 3)];
(e) x(t) = 1/t;
(f) x(t) = t rect(2t);
(g) x(t) = e− j3t sin(5t − 2);
(h) x(t) = cos(5t − 2);
(i) x(t) = e− j2t 3t+1
1
;
R 5t −τ −1
(j) x(t) = −∞ e u(τ − 1)d τ ;
(k) x(t) = (t + 1) sin(5t − 3);
(l) x(t) = (sin 2π t)δ (t − π2 );
(m) x(t) = e− jt 3t−2
1
;
j5t
(n) x(t) = e (cos 2t)u(t); and
(o) x(t) = e− j2t sgn(−t − 1).
Solution.
(c) We begin by rewriting x(t) as

x(t) = v1 (t)v2 (t),

where

v1 (t) = cost and


v2 (t) = u(t).

Copyright 2014
c Michael D. Adams Version: 2014-05-08
9

Taking the Fourier transform of both sides of each of the above equations yields

X(ω ) = 2π V1 (ω ) ∗V2 (ω ),
1

V1 (ω ) = π [δ (ω − 1) + δ (ω + 1)], and
V2 (ω ) = πδ (ω ) + 1
jω .

Combining the above results, we obtain

X(ω ) = 2π V1 (ω ) ∗V2 (ω )
1
Z ∞
= 1
2π V1 (λ )V2 (ω − λ )d λ
−∞
Z ∞ h i
= 1
2π π [δ (λ − 1) + δ (λ + 1)] πδ (ω − λ ) + j(ω1−λ ) d λ
−∞
Z ∞ h i
= 1
2π π πδ (λ − 1)δ (ω − λ ) + δ (λ − 1) j(ω1−λ ) + πδ (λ + 1)δ (ω − λ ) + δ (λ + 1) j(ω1−λ ) d λ
−∞
h i
= 1
2 πδ (ω − 1) + j(ω1−1) + πδ (ω + 1) + j(ω1+1)
h i
= 12 πδ (ω − 1) + πδ (ω + 1) − ω −1j j
− ω +1

= 21 πδ (ω − 1) + πδ (ω + 1) + − j(ω −1)− j(ω +1)


h i
ω −1
2

j2ω
h i
= 2 πδ (ω − 1) + πδ (ω + 1) − ω 2 −1
1

π jω
= [δ (ω − 1) + δ (ω + 1)] − .
2 ω2 − 1

(d) We begin by rewriting x(t) as

x(t) = 6v3 (t),

where

v3 (t) = v2 (t/3),
v2 (t) = v1 (t − 12 ), and
v1 (t) = rect(t).

Taking the Fourier transform of both sides of each of the above equations yields

X(ω ) = 6V3 (ω ),
V3 (ω ) = 3V2 (3ω ),
V2 (ω ) = e− jω /2V1 (ω ), and
V1 (ω ) = sinc ω /2.

Combining the above results, we have

X(ω ) = 6V3 (ω )
= 6(3)V2 (3ω )
= 18V2 (3ω )
= 18e− j3ω /2V1 (3ω )
= 18e− j3ω /2 sinc 32ω .

Version: 2014-05-08 Copyright 2014


c Michael D. Adams
10 Continuous-Time Fourier Transform (Chapter 5)

Alternatively, we can restate this result in a slightly different form (i.e., in terms of complex exponentials) as
follows:

X(ω ) = 18e− j3ω /2 sinc 32ω


= 18e− j3ω /2 32ω 21j e j3ω /2 − e− j3ω /2
h h ii

6 − j3ω
= jω [1 − e ].

A LTERNATIVE SOLUTION . We have

X(ω ) = 6 (F {u(t)} − F {u(t − 3)})


= 6 πδ (ω ) + j1ω − e− j3ω (πδ (ω ) + j1ω )
 

= 6 πδ (ω ) + j1ω − πδ (ω )e− j3ω − j1ω e− j3ω


 

= 6 πδ (ω ) + j1ω − πδ (ω ) − j1ω e− j3ω


 

6 − j3ω
= jω (1 − e )
6 − j3ω /2 j3ω /2
= jω e (e − e− j3ω /2 )
6 − j3ω /2
= jω e (2 j) sin 3ω /2
12 − j3ω /2
= ωe sin 3ω /2
3ω 12 − j3ω /2 3ω −1
= 2 ωe ( 2 ) sin 3ω /2
− j3ω /2
= 18e sinc 3ω /2.

(e) From a table of Fourier transforms, we have


F 2
sgnt ←→ jω .

From this transform pair, we can use the duality property of the Fourier transform to deduce

F { 2jt } = 2π sgn(−ω )
= −2π sgn ω .

Using this result and the linearity property of the Fourier transform, we can write

X(ω ) = F {1/t}
= 2j F { 2jt }
= 2j [−2π sgn ω ]
= − jπ sgn ω .

(f) We begin by rewriting x(t) as

x(t) = tv2 (t),

where

v2 (t) = v1 (2t) and


v1 (t) = rect(t).

Copyright 2014
c Michael D. Adams Version: 2014-05-08
11

Taking the Fourier transform of both sides of each of the above equations, we obtain

V1 (ω ) = sinc ω /2,
V2 (ω ) = 12 V1 ( ω2 ), and
X(ω ) = j ddω V2 (ω ).

Combining the above results, we have

X(ω ) = j ddω V2 (ω )
= j ddω [ 21 V1 ( ω2 )]
j d ω
= 2 d ω V1 ( 2 )
j d ω
= 2 d ω sinc 4
ω 1 ω
sin ω4
" #
1
j 4 ( 4 cos 4 ) − 4
= 2 ω 2 /16
ω
cos ω4 − 14 sin ω4 )
" #
j 16( 16
= 2 ω2

= 2j ω1 cos ω4 − ω42 sin ω4


h i

= j
2ω cos ω4 − ωj22 sin ω4 .

(g) We begin by rewriting x(t) as

x(t) = e− j3t v3 (t),

where

v3 (t) = v2 (5t),
v2 (t) = v1 (t − 2), and
v1 (t) = sint.

Taking the Fourier transform of both sides of each of the above equations yields
π
V1 (ω ) = j [δ (ω − 1) − δ (ω + 1)] ,
V2 (ω ) = e− j2ω V1 (ω ),
V3 (ω ) = 15 V2 ( ω5 ), and
X(ω ) = V3 (ω + 3).

Combining the above results, we obtain

X(ω ) = V3 (ω + 3)
= 51 V2 ( ω +3
5 )
= 15 e− j2(ω +3)/5V1 ( ω +3
5 )
π ω
− j2( +3)/5
δ ( ω +3 ω +3
5 − 1) − δ ( 5 + 1)
 
= j5 e
= − j5π e− j2 δ ( ω −2
5 )+
jπ j2 ω +8
5 e δ ( 5 ).

(In the above simplification, we used the relationship f (t)δ (at − b) = f (b/a)δ (at − b). This identity follows
immediately from the fact that δ (at − b) = 0 for all t 6= b/a.)

Version: 2014-05-08 Copyright 2014


c Michael D. Adams
12 Continuous-Time Fourier Transform (Chapter 5)

F F
5.5 Given that x(t) ←→ X(ω ) and y(t) ←→ Y (ω ), express Y (ω ) in terms of X(ω ) for each of the following:
(a) y(t) = x(at − b) where a and b are constants and a 6= 0;
x(τ )d τ ;
R 2t
(b) y(t) = −∞
x (τ )d τ ;
Rt 2
(c) y(t) = −∞
(d) y(t) = dtd [x(t) ∗ x(t)];
(e) y(t) = tx(2t − 1);
(f) y(t) = e j2t x(t − 1);

(g) y(t) = te− j5t x(t) ;
d   − jt 
(h) y(t) = dt x(t) ∗ e x(t) ;
x (τ − 1)d τ ;
R 3t ∗
(i) y(t) = −∞
(j) y(t) = [cos(3t −
 1)] x(t);
(k) y(t) = dtd x(t) sin(t − 2);


(l) y(t) = tx(t) sin 3t; and


(m) y(t) = e j7t [x(λ ) ∗ x(λ )]|λ =t−1 .
Solution.
(a) We rewrite y(t) as

y(t) = v1 (at)

where

v1 (t) = x(t − b).

Taking the Fourier transform of both sides of the above equations yields
ω
Y (ω ) = 1
|a| V1 ( a ) and
− jω b
V1 (ω ) = e X(ω ).

Combining these equations, we obtain


ω
Y (ω ) = 1
|a| V1 ( a )
1 − j(ω /a)b
= |a| e X(ω /a)
1 − jbω /a
= |a| e X(ω /a).

(b) We rewrite y(t) as

y(t) = v1 (2t)

where
Z t
v1 (t) = x(τ )d τ .
−∞

Taking the Fourier transform of both sides of the above equations yields

Y (ω ) = F {v1 (2t)}
= 21 V1 ( ω2 ) and
Z t 
V1 (ω ) = F x(τ )d τ
−∞

jω X(ω ) + π X(0)δ (ω ).
1
=

Copyright 2014
c Michael D. Adams Version: 2014-05-08
13

Combining the above equations, we obtain

Y (ω ) = 21 V1 ( ω2 )
 
1 ω ω
=21
X( ) + π X(0)δ ( 2 )
j(ω /2) 2
ω π ω
jω X( 2 ) + 2 X(0)δ ( 2 ).
1
=

(c) We rewrite y(t) as


Z t
y(t) = v1 (τ )d τ
−∞

where

v1 (t) = x2 (t).

Taking the Fourier transform of both sides of each of the above equations yields

V1 (ω ) = 2π X(ω ) ∗ X(ω ), and


1

Y (ω ) = jω V1 (ω ) + π V1 (0)δ (ω ).
1

Combining the above results, we have


 Z∞   Z ∞

Y (ω ) = jω 2π
1 1
X(λ )X(ω − λ )d λ + π 21π X(λ )X(−λ )d λ δ (ω )
−∞ −∞
Z ∞ Z ∞
= 1
j2πω X(λ )X(ω − λ )d λ + 12 δ (ω ) X(λ )X(−λ )d λ .
−∞ −∞

(d) We rewrite y(t) as


d
y(t) = dt v1 (t)

where

v1 (t) = x(t) ∗ x(t).

Taking the Fourier transform of both sides of these equations yields

Y (ω ) = F { dtd v1 (t)}
= jω V1 (ω ) and
V1 (ω ) = F {x(t) ∗ x(t)}
= X 2 (ω ).

Combining these equations, we obtain

Y (ω ) = jω V1 (ω )
= jω X 2 (ω ).

Version: 2014-05-08 Copyright 2014


c Michael D. Adams
14 Continuous-Time Fourier Transform (Chapter 5)

(e) We rewrite y(t) as

y(t) = tv1 (t),

where

v1 (t) = v2 (2t) and


v2 (t) = x(t − 1).

Taking the Fourier transform of both sides of the above equations yields

Y (ω ) = F {tv1 (t)}
= j ddω V1 (ω ),
V1 (ω ) = F {v2 (2t)}
= 21 V2 ( ω2 ), and
V2 (ω ) = F {x(t − 1)}
= e− jω X(ω ).

Combining these equations, we obtain

Y (ω ) = j ddω V1 (ω )
= j ddω ( 21 )V2 ( ω2 )
 

= 2j ddω e− jω /2 X( ω2 ) .
h i

A LTERNATE S OLUTION . In what follows, we use the prime symbol to denote derivative (i.e., f ′ denotes the
derivative of f ). We can rewrite y(t) as

y(t) = tv1 (t),

where

v1 (t) = v2 (2t), and


v2 (t) = x(t − 1).

Taking the Fourier transform of both sides of the above equations, we obtain

Y (ω ) = jV1′ (ω ),
V1 (ω ) = 21 V2 (ω /2), and
V2 (ω ) = e− jω X(ω ).

In anticipation of what is to come, we compute the quantities:

V1′ (ω ) = 21 21 V2′ (ω /2) = 14 V2′ (ω /2) and


 

V2′ (ω ) = − je− jω X(ω ) + X ′ (ω )e− jω .

Combining the above equations, we have

Y (ω ) = jV1′ (ω )
= j 41 V2′ (ω /2)
= 4j − je− jω /2 X(ω /2) + e− jω /2 X ′ (ω /2) .
h i

Copyright 2014
c Michael D. Adams Version: 2014-05-08
15

(f) We begin by rewriting y(t) as

y(t) = e j2t v1 (t)

where

v1 (t) = x(t − 1).

Taking the Fourier transform of both sides of the above equations yields

V1 (ω ) = e− jω X(ω ) and
Y (ω ) = V1 (ω − 2).

Combining the above results, we have

Y (ω ) = V1 (ω − 2)
= e− j(ω −2) X(ω − 2).

5.6 Find the Fourier transform of each of the periodic signals shown below.

x(t)
1 1
x(t)
1
··· ···
t ··· ···
−4 −3 −2 −1 1 2 3 4

t
−4 −3 −2 −1 0 1 2 3 4

−1 −1 (b)

(a)

Solution.

(a) The frequency ω0 is given by ω0 = T = π2 . Consider the period of x(t) for −2 ≤ t < 2. Let us denote this
single period as xT (t). We have

xT (t) = −δ (t + 1) + δ (t − 1).

Taking the Fourier transform of xT (t), we obtain

XT (ω ) = F {δ (t − 1) − δ (t + 1)}
= F {δ (t − 1)} − F {δ (t + 1)}
= e− jω − e jω
= −[e jω − e− jω ]
= −2 j sin ω .

Version: 2014-05-08 Copyright 2014


c Michael D. Adams
16 Continuous-Time Fourier Transform (Chapter 5)

Using the formula for the Fourier transform of a periodic signal, we obtain

X(ω ) = F {x(t)}

= ∑ ω0 XT (kω0 )δ (ω − kω0 )
k=−∞

= ∑ π π π
2 [−2 j sin k 2 ]δ (ω − k 2 )
k=−∞

= ∑ − jπ (sin k2π )δ (ω − k2π ).
k=−∞

5.9 Compute the frequency spectrum of each of the signals specified below. In each case, also find and plot the
corresponding magnitude and phase spectra.
(a) x(t) = e−at u(t), where a is a positive real constant; and
(b) x(t) = sinc t−1
200 .

Solution.

(a) Taking the Fourier transform of x(t), we obtain

X(ω ) = F {e−at u(t)}


1
= .
a + jω

Computing the magnitude spectrum, we obtain


1
|X(ω )| =

a + jω
|1|
=
|a + jω |
1
=√ .
a + ω2
2

Computing the phase spectrum, we obtain

 
1
arg X(ω ) = arg
a + jω
= arg 1 − arg(a + jω )
= 0 − arg(a + jω )
= − arg(a + jω )
= − arctan ωa .

The magnitude and phase spectra are plotted below for a = 1.

Copyright 2014
c Michael D. Adams Version: 2014-05-08
17

1 1.5

0.9

1
0.8

0.7
0.5
Magnitude 0.6

Phase
0.5 0

0.4

−0.5
0.3

0.2
−1

0.1

0 −1.5
−10 −8 −6 −4 −2 0 2 4 6 8 10 −10 −8 −6 −4 −2 0 2 4 6 8 10
Frequency Frequency

(a) (b)
5.10 Suppose that we have the LTI systems defined by the differential/integral equations given below, where x(t) and
y(t) denote the system input and output, respectively. Find the frequency response of each of these systems.
2
(a) dtd 2 y(t) + 5 dtd y(t) + y(t) + 3 dtd x(t) − x(t) = 0; and
3y(τ )d τ + 5 dtd x(t) − x(t) = 0.
Rt
(b) dtd y(t) + 2y(t) + −∞
Solution.
(b) First, we differentiate the given equation with respect to t. This yields
( dtd )2 y(t) + 2 dtd y(t) + 3y(t) + 5( dtd )2 x(t) − dtd x(t) = 0.
Taking the Fourier transform of both sides of the above equation, we obtain
F {( dtd )2 y(t) + 2 dtd y(t) + 3y(t) + 5( dtd )2 x(t) − dtd x(t)} = 0
⇒ F {( dtd )2 y(t)} + 2F { dtd y(t)} + 3F {y(t)} + 5F {( dtd )2 x(t)} − F { dtd x(t)} = 0
⇒ ( jω )2Y (ω ) + 2( jω )Y (ω ) + 3Y (ω ) + 5( jω )2 X(ω ) − ( jω )X(ω ) = 0
⇒ − ω 2Y (ω ) + j2ω Y (ω ) + 3Y (ω ) − 5ω 2 X(ω ) − jω X(ω ) = 0
⇒ [−ω 2 + j2ω + 3]Y (ω ) = [5ω 2 + jω ]X(ω ).
Therefore, the frequency response H(ω ) of the system is
Y (ω ) 5ω 2 + jω
H(ω ) = = .
X(ω ) −ω 2 + 2 jω + 3

5.11 Suppose that we have the LTI systems with the frequency responses given below. Find the differential equation
that characterizes each of these systems.

(a) H(ω ) = ; and
1 + jω
jω + 21
(b) H(ω ) = .
− jω 3 − 6ω 2 + 11 jω + 6
Solution.
(b) From the given frequency response H(ω ), we can write
Y (ω ) jω + 12
=
X(ω ) − jω − 6ω 2 + 11 jω + 6
3

⇒ [− jω 3 − 6ω 2 + 11 jω + 6]Y (ω ) = [ jω + 21 ]X(ω )
⇒ − jω 3Y (ω ) − 6ω 2Y (ω ) + 11 jω Y (ω ) + 6Y (ω ) = jω X(ω ) + 12 X(ω )
⇒ − jω 3Y (ω ) − 6ω 2Y (ω ) + 11 jω Y (ω ) + 6Y (ω ) − jω X(ω ) − 21 X(ω ) = 0.

Version: 2014-05-08 Copyright 2014


c Michael D. Adams
18 Continuous-Time Fourier Transform (Chapter 5)

Taking the inverse Fourier transform of both sides of the above equation, we obtain
F −1 {− jω 3Y (ω ) − 6ω 2Y (ω ) + 11 jω Y (ω ) + 6Y (ω ) − jω X(ω ) − 21 X(ω )} = 0
⇒ F −1 {( jω )3Y (ω )} + 6F −1 {( jω )2Y (ω )} + 11F −1 { jω Y (ω )} + 6F −1 {Y (ω )}
− F −1 { jω X(ω )} − 21 F −1 {X(ω )} = 0
⇒ ( dtd )3 y(t) + 6( dtd )2 y(t) + 11 dtd y(t) + 6y(t) − dtd x(t) − 12 x(t) = 0.

5.12 Suppose that we have a LTI system with input x(t) and output y(t), and impulse response h(t), where
h(t) = δ (t) − 300 sinc 300π t.
Using frequency-domain methods, find the response y(t) of the system to the input x(t) = x1 (t), where
x1 (t) = 12 + 43 cos 200π t + 12 cos 400π t − 14 cos 600π t.

Solution.
Let X(ω ), Y (ω ), and H(ω ) denote the Fourier transforms of x(t), y(t), and h(t), respectively.
We begin by finding the frequency response H(ω ) of the system. This process yields
π
H(ω ) = F {δ (t)} − F { 300
π sinc 300π t}
ω
= 1 − rect( 600π )
(
1 for |ω | > 300π
=
0 otherwise.

Next, we determine the Fourier transform X1 (ω ) of the input signal. We have


X1 (ω ) = F { 12 } + 43 F {cos 200π t} + 12 F {cos 400π t} − 14 F {cos 600π t}
= πδ (ω ) + 43 π [δ (ω − 200π ) + δ (ω + 200π )] + 12 π [δ (ω − 400π ) + δ (ω + 400π )]
− 41 π [δ (ω − 600π ) + δ (ω + 600π )].
Since the system is LTI, we can write
Y (ω ) = X1 (ω )H(ω )
= 21 π [δ (ω − 400π ) + δ (ω + 400π )] − 41 π [δ (ω − 600π ) + δ (ω + 600π )].
Taking the inverse Fourier transform of Y (ω ), we obtain
y(t) = F −1 {Y (ω )}
= 12 F −1 {π [δ (ω − 400π ) + δ (ω + 400π )]} − 41 F −1 {π [δ (ω − 600π ) + δ (ω + 600π )]}
= 12 cos 400π t − 14 cos 600π t.

5.13 Consider the LTI system with input v0 (t) and output v1 (t) as shown in the figure below, where R = 1 and L = 1.
R
i(t)
+ +

v0 (t) L v1 (t)

− −

Copyright 2014
c Michael D. Adams Version: 2014-05-08
19

(a) Find the frequency response H(ω ) of the system.


(b) Determine the magnitude and phase responses of the system.
(c) Find the impulse response h(t) of the system.
Solution.
(a) From basic circuit analysis, we can write
v1 (t) = L dtd [ R1 [v0 (t) − v1 (t)]]
L d L d
⇒ v1 (t) = R dt v0 (t) − R dt v1 (t)

Taking the Fourier transform of the preceding equation, we have


V1 (ω ) = RL F { dtd v0 (t)} − RL F { dtd v1 (t)}
⇒ V1 (ω ) = L
R jω V0 (ω ) − RL jω V1 (ω )
⇒ [1 + RL jω ]V1 (ω ) = L
R jω V0 (ω )

V1 (ω )
H(ω ) =
V0 (ω )
L
Rjω
=
1 + RL jω

= 1+ jω .

(b) Taking the magnitude of the frequency response H(ω ), we obtain


| jω |
|H(ω )| =
|1 + jω |
|ω |
=√ .
1 + ω2
Taking the argument of the frequency response H(ω ), we obtain
arg H(ω ) = arg jω − arg(1 + jω )
π
= 2 sgn ω − tan−1 ω .
As an aside, we note that
π
(
for ω > 0
arg jω = 2
− π2 for ω < 0
π
= 2 sgn ω .

(c) Taking the inverse Fourier transform of the frequency response H(ω ) yields
h(t) = F −1 { jω ( 1+1jω )}
d −1
= dt F { 1+1jω }
d −t
= dt [e u(t)]
= −e u(t) + δ (t)e−t
−t

= −e−t u(t) + δ (t).

Version: 2014-05-08 Copyright 2014


c Michael D. Adams
20 Continuous-Time Fourier Transform (Chapter 5)

5.18 Consider the system shown below in Figure A with input x(t) and output x̂(t), where
(
2 for |ω | ≤ 100π
G(ω ) =
0 otherwise.

Let X(ω ), X̂(ω ), Y (ω ), and Q(ω ) denote the Fourier transforms of x(t), x̂(t), y(t), and q(t), respectively.

X1 (ω )
sin 1000π t sin 1000π t
1

x(t) y(t) q(t) x̂(t)


× × G(ω )

(a) ω
−100π 0 100π
(b)

(a) Suppose that X(ω ) = 0 for |ω | > 100π . Find expressions for Y (ω ), Q(ω ), and X̂(ω ) in terms of X(ω ).
(b) If X(ω ) = X1 (ω ) where X1 (ω ) is as shown in Figure B, sketch Y (ω ), Q(ω ), and X̂(ω ).
Solution.
(a) From the system block diagram, we have
Y (ω ) = F {x(t) sin 1000π t}
= F { 21j [e j1000π t − e− j1000π t ]x(t)}
j1000π t
= 1
2 j F {e x(t)} − 21j F {e− j1000π t x(t)}

2 j X(ω − 1000π ) − 2 j X(ω + 1000π ),


1 1
= (4.1)
Q(ω ) = F {y(t) sin 1000π t}
= F { 21j [e j1000π t − e− j1000π t ]y(t)}
j1000π t
= 1
2 j F {e y(t)} − 21j F {e− j1000π t y(t)}

2 j Y (ω − 1000π ) − 2 j Y (ω + 1000π ), and


1 1
= (4.2)
X̂(ω ) = G(ω )Q(ω ). (4.3)
Substituting the expression for Y (ω ) from (4.1) into (4.2), we have
Q(ω ) = 2 j Y (ω − 1000π ) − 2 j Y (ω + 1000π )
1 1

= 21j [ 21j X([ω − 1000π ] − 1000π ) − 21j X([ω − 1000π ] + 1000π )]


− 21j [ 21j X([ω + 1000π ] − 1000π ) − 21j X([ω + 1000π ] + 1000π )]
= − 14 X(ω − 2000π ) + 41 X(ω ) + 41 X(ω ) − 14 X(ω + 2000π )
= 21 X(ω ) − 14 X(ω − 2000π ) − 41 X(ω + 2000π ). (4.4)
Combining (4.3) and (4.4) and using the fact that X(ω ) = 0 for |ω | > 100π , we have
X̂(ω ) = G(ω )Q(ω )
= 2( 21 X(ω ))
= X(ω ).

(b) The frequency spectra of the various signals are plotted below, where ωc = 1000π and ωb = 100π .

Copyright 2014
c Michael D. Adams Version: 2014-05-08
21

Y (ω )
j
2

ω
−2ωc −ω c − ω b −ω c −ωc + ωb −ω b ωb ωc − ωb ωc ωc + ωb 2ωc

− 2j

(a)
Q(ω )
1
2

1
4

ω
−2ωc − ωb −2ωc −2ωc + ωb −ω b ωb 2ωc − ωb 2ωc 2ωc + ωb

− 41

(b)
X̂(ω )

ω
−2ωc −ω c −ω b ωb ωc 2ωc

(c)

5.19 Consider the system shown below in Figure A with input x(t) and output y(t). Let X(ω ), P(ω ), S(ω ), H(ω ),
and Y (ω ) denote the Fourier transforms of x(t), p(t), s(t), h(t), and y(t), respectively. Suppose that


p(t) = ∑ δ (t − 1000
n
) and H(ω ) = 1
1000
ω
rect( 2000 π ).
n=−∞

(a) Derive an expression for S(ω ) in terms of X(ω ). Derive an expression for Y (ω ) in terms of S(ω ) and H(ω ).
(b) Suppose that X(ω ) = X1 (ω ), where X1 (ω ) is as shown in Figure B. Using the results of part (a), plot S(ω )
and Y (ω ). Indicate the relationship (if any) between the input x(t) and output y(t) of the system.
(c) Suppose that X(ω ) = X2 (ω ), where X2 (ω ) is as shown in Figure C. Using the results of part (a), plot S(ω )
and Y (ω ). Indicate the relationship (if any) between the input x(t) and output y(t) of the system.

p(t)

x(t) s(t) y(t)


× h(t)

(a)

Version: 2014-05-08 Copyright 2014


c Michael D. Adams
22 Continuous-Time Fourier Transform (Chapter 5)

X1 (ω ) X2 (ω )

1 1

ω ω
−1000π 1000π −2000π 2000π
(b) (c)

Solution.
(a) Since p(t) is periodic with period T = 1
1000 (and frequency ω0 = 2000π ), it can be expressed in terms of a
Fourier series as

p(t) = ∑ ck e j2000π kt .
k=−∞

Using the Fourier series analysis equation, we compute ck as


Z 1/2000
δ (t)e− j2000π kt dt
1
−1
ck = 1000
−1/2000
= 1000.
Combining the above equations, we have

p(t) = 1000 ∑ e j2000π kt .
k=−∞

From the system block diagram, we have


s(t) = x(t)p(t)

!
= x(t) 1000 ∑ e j2000π kt
k=−∞

= 1000 ∑ x(t)e j2000π kt .
k=−∞

Taking the Fourier transform of both sides of the preceding equation, we obtain

S(ω ) = 1000 ∑ X(ω − 2000π k).
k=−∞

From the system block diagram, we have


y(t) = s(t) ∗ h(t).
Taking the Fourier transform of both sides of the preceding equation, we obtain
Y (ω ) = H(ω )S(ω )
(
1
S(ω ) for |ω | < 1000π
= 1000
0 otherwise
 ∞
∑ X(ω − 2000π k) for |ω | < 1000π


= k=−∞

0 otherwise.

Copyright 2014
c Michael D. Adams Version: 2014-05-08
23

(b) We observe that X(ω ) = 0 for |ω | < 1000π . Therefore, the copies of the original spectrum of X(ω ) in S(ω )
do not overlap. A plot of S(ω ) is provided below. The plot of Y (ω ) is identical to that of X(ω ) = X1 (ω ) given
in the problem statement.

S(ω )

1000

ω
−1000π 0 1000π

Since X(ω ) = Y (ω ), the input and output of the system are identical.
(c) We observe that X(ω ) 6= 0 for some ω satisfying |ω | > 1000π . Therefore, the copies of the original spectrum
in S(ω ) overlap, resulting in aliasing. The plots of S(ω ) and Y (ω ) are given below.

S(ω ) Y (ω )

1000 1

ω ω
−1000π 0 1000π −1000π 0 1000π

We can see that Y (ω ) does not in any way resemble X(ω ). The input x(t) and output y(t) are not related, except
in the sense that the output is an extremely distorted version of the input, with the distortion being caused by
aliasing.
5.101 (a) Consider a frequency response H(ω ) of the form

∑k=0
M−1
ak ω k
H(ω ) = ,
∑k=0 bk ω k
N−1

where ak and bk are complex constants. Write a MATLAB function called freqw that evaluates a function of
the above form at an arbitrary number of specified points. The function should take three input arguments: 1) a
vector containing the ak coefficients, 2) a vector containing the bk coefficients, 3) a vector containing the values
of ω at which to evaluate H(ω ). The function should generate two return values: 1) a vector of function values,
and 2) a vector of points at which the function was evaluated. If the function is called with no output arguments
(i.e., the nargout variable is zero), then the function should plot the magnitude and phase responses before
returning. [Hint: The polyval function may be helpful.]
(b) Use the function developed in part (a) to plot the magnitude and phase responses of the system with the
frequency response
16.0000
H(ω ) = .
1.0000ω 4 − j5.2263ω 3 − 13.6569ω 2 + j20.9050ω + 16.0000
For each of the plots, use the frequency range [−5, 5].
(c) What type of ideal frequency-selective filter does this system approximate?
Solution.
(a) The freqw function can be implemented with the code below.

Version: 2014-05-08 Copyright 2014


c Michael D. Adams
24 Continuous-Time Fourier Transform (Chapter 5)

Listing 4.1: freqw.m

function [ freqresp , omega ] = freqw ( ncoefs , dcoefs , omega )

freqresp = polyval ( ncoefs , omega ) ./ polyval ( dcoefs , omega );

% If no output arguments were specified, plot the frequency response.


if nargout == 0

% Compute the magnitude response as a unitless quantity.


magresp = abs ( freqresp );

% Compute the phase response.


phaseresp = angle ( freqresp );

% On the first of two graphs, plot the magnitude response.


subplot (2 , 1, 1);
plot ( omega , magresp );
title ( ’ Magnitude Response ’);
xlabel ( ’ Frequency ( rad /s) ’);
ylabel ( ’ Magnitude ( unitless ) ’);

% On the second of two graphs, plot the phase response.


subplot (2 , 1, 2);
plot ( omega , phaseresp );
title ( ’ Phase Response ’);
xlabel ( ’ Frequency ( rad /s) ’);
ylabel ( ’ Angle ( rad ) ’);

end

Using the freqw function, we can generate the necessary plots with the following few lines of code:

ncoefs = [16];
dcoefs = [1.0000 (-j * 5.2263) ( -13.6569) (j * 20.9050) 16.0000];
freqw ( ncoefs , dcoefs , linspace ( -5 , 5, 500));

(b) The magnitude and phase responses are shown in the figure below.

Copyright 2014
c Michael D. Adams Version: 2014-05-08
25

Magnitude Response
1.4

1.2

Magnitude (unitless)
1

0.8

0.6

0.4

0.2

0
−5 −4 −3 −2 −1 0 1 2 3 4 5
Frequency (rad/s)

Phase Response
4

2
Angle (rad)

−2

−4
−5 −4 −3 −2 −1 0 1 2 3 4 5
Frequency (rad/s)

(c) The system approximates a lowpass filter with a cutoff frequency somewhere in the vicinity of 2 rad/s.

5.103 (a) Use the butter and besself functions to design a tenth-order Butterworth lowpass filter and tenth-order
Bessel lowpass filter, each with a cutoff frequency of 10 rad/s.
(b) For each of the filters designed in part (a), plot the magnitude and phase responses using a linear scale for
the frequency axis. In the case of the phase response, plot the unwrapped phase (as this will be helpful later in
part (d) of this problem). (Hint: The freqs and unwrap functions may be helpful.)
(c) Consider the magnitude responses for each of the filters. Recall that an ideal lowpass filter has a magnitude
response that is constant in the passband. Which of the two filters more closely approximates this ideal behav-
ior?
(d) Consider the phase responses for each of the filters. An ideal lowpass filter has a phase response that is a
linear function. Which of the two filters has a phase response that best approximates a linear (i.e., straight line)
function in the passband?
Solution.
(a) and (b) The frequency responses for the two filters can be plotted using the code shown below.
% Choose a filter type.
filtertype = ’ butterworth ’;
% filtertype = ’bessel’;

% Set the cutoff frequency for the filter.


wc = 10;

% Calculate the transfer function coefficients for the filter.


switch filtertype
case { ’ butterworth ’}
[ tfnum , tfdenom ] = butter (10 , wc , ’s ’);
case { ’ bessel ’}
[ tfnum , tfdenom ] = besself (10 , wc );
end

% Calculate the magnitude and phase responses.


[ freqresp , omega ] = freqs ( tfnum , tfdenom , linspace ( -20 , 20 , 500));
magresp = abs ( freqresp );

Version: 2014-05-08 Copyright 2014


c Michael D. Adams
26 Continuous-Time Fourier Transform (Chapter 5)

phaseresp = unwrap ( angle ( freqresp ));

% Plot the magnitude and phase response (using the unwrapped phase).
clf
subplot (2 , 1, 1);
plot ( omega , magresp );
title ( ’ Magnitude Response ’);
xlabel ( ’ Frequency ( rad /s) ’);
ylabel ( ’ Magnitude ( unitless ) ’);
subplot (2 , 1, 2);
plot ( omega , phaseresp );
title ( ’ Phase Response ’);
xlabel ( ’ Frequency ( rad /s) ’);
ylabel ( ’ Angle ( rad ) ’);

The above code produces the plots shown below.


Magnitude Response
Magnitude Response
1
1.2

1 0.8

Magnitude (unitless)
Magnitude (unitless)

0.8 0.6

0.6
0.4
0.4
0.2
0.2
0
0 −20 −15 −10 −5 0 5 10 15 20
−20 −15 −10 −5 0 5 10 15 20
Frequency (rad/s)
Frequency (rad/s)
Phase Response
Phase Response
0
0

−5
−5
Angle (rad)
Angle (rad)

−10
−10

−15
−15

−20
−20

−25
−25 −20 −15 −10 −5 0 5 10 15 20
−20 −15 −10 −5 0 5 10 15 20
Frequency (rad/s)
Frequency (rad/s)

(a) (b)
(c) In the passband, the magnitude response of the Butterworth filter is much flatter than that of the Bessel filter.
As it turns out, Butterworth filters have very flat magnitude responses in the passband.
(d) The Bessel filter has a phase response that is closer to having linear phase than the Butterworth filter. As it
turns out, Bessel filters tend to have phase responses that are approximately linear in the passband.

Copyright 2014
c Michael D. Adams Version: 2014-05-08
37

Chapter 10

Review (Appendix F)

F.1 A communication channel heavily distorts high frequencies but does not significantly affect very low frequen-
cies. Determine which of the following signals would be least distorted by the communication channel:
(a) x1 (t) = δ (t)
(b) x2 (t) = 5
(c) x3 (t) = 10e j1000t
(d) x4 (t) = 1/t
Answer:
x2 (t)
Solution. Let X1 (ω ), X2 (ω ), X3 (ω ), and X4 (ω ) denote the Fourier transforms of x1 (t), x2 (t), x3 (t), and x4 (t),
respectively. Taking the Fourier transforms of x1 (t), x2 (t), x3 (t), and x4 (t), we obtain:

X1 (ω ) = 1,
X2 (ω ) = 5(2πδ (ω ))
= 10πδ (ω ),
X3 (ω ) = 10(2πδ (ω − 1000))
= 20πδ (ω − 1000), and
X4 (ω ) = − jπ sgn ω .

From the above Fourier transforms, we can see that the signals x1 (t), x3 (t), and x4 (t), all contain information at
relatively high frequencies, while the signal x2 (t) only has information at the frequency 0. Therefore, x2 (t) will
be least distorted. Note that, in computing X4 (ω ), we used the fact that
F 2
sgnt ←→ jω
F
⇔ 2
jt ←→ 2π sgn(−ω )
F
⇔ 1/t ←→ jπ sgn(−ω ) = − jπ sgn ω .

F.37 A signal x(t) is bandlimited to 22 kHz (i.e., only has spectral content for frequencies f in the range [−22000, 22000]).
Due to excessive noise, the portion of the spectrum that corresponds to frequencies f satisfying | f | > 20000 has
been badly corrupted and rendered useless. (a) Determine the minimum sampling rate for x(t) that would allow
the uncorrupted part of the spectrum to be recovered. (b) Suppose now that the corrupted part of the spectrum
were eliminated by filtering prior to sampling. In this case, determine the minimum sampling rate for x(t).
Answer:

Version: 2014-05-08 Copyright 2014


c Michael D. Adams
38 Review (Appendix F)

(a) 42 kHz; (b) 40 kHz


Solution. Let y(t) denote the signal obtained from x(t) after sampling and reconstruction. Let X(ω ) and Y (ω )
denote the Fourier transforms of x(t) and y(t), respectively. The spectrum X(ω ) has a form something like that
shown in the figure below.

X(ω )

corrupted corrupted

ω
−44000π 44000π
−40000π 40000π

(a) Since we only wish to be able to recover the uncorrupted part of the spectrum of x(t), it does not matter if
aliasing occurs in the range of frequencies where the spectrum has already been corrupted. If we choose the
sampling rate to be 84000π , we obtain the spectrum shown in the figure below, after impulse sampling.

Y (ω )

aliasing aliasing

ω
−84000π −44000π 44000π 84000π
−40000π 40000π

From this plot, we can see that aliasing only occurs in the corrupted part of the spectrum. Thus, we can recover
the uncorrupted part of the spectrum by lowpass filtering. Using a lower sampling rate would cause aliasing to
occur in the uncorrupted part of the spectrum. Thus, the minimum sampling rate required is 84000π rad/s (or
equivalently, 42 kHz).
(b) Since the corrupted part of the spectrum has been removed (i.e., set to zero), the new signal is bandlimited
to frequencies in the range [−40000π , 40000π ]. From the sampling theorem, we must sample the signal at

ωs = 2(40000π )
= 80000π .

Thus, a sampling rate of 80000π rad/s (or equivalently, 40 kHz) is required.

Copyright 2014
c Michael D. Adams Version: 2014-05-08

Você também pode gostar